Mathcenter Forum  

Go Back   Mathcenter Forum > คณิตศาสตร์โอลิมปิก และอุดมศึกษา > ทฤษฎีจำนวน
สมัครสมาชิก คู่มือการใช้ รายชื่อสมาชิก ปฏิทิน ข้อความวันนี้

ตั้งหัวข้อใหม่ Reply
 
เครื่องมือของหัวข้อ ค้นหาในหัวข้อนี้
  #241  
Old 21 เมษายน 2015, 19:26
ความรู้ยังอ่อนด้อย's Avatar
ความรู้ยังอ่อนด้อย ความรู้ยังอ่อนด้อย ไม่อยู่ในระบบ
จอมยุทธ์หน้าหยก
 
วันที่สมัครสมาชิก: 18 กันยายน 2010
ข้อความ: 175
ความรู้ยังอ่อนด้อย is on a distinguished road
Default

อ้างอิง:
ข้อความเดิมเขียนโดยคุณ Beatmania View Post
ลองดูครับว่าถ้าหาก $n$ ไม่อยู่ในรูป $\left\lfloor b\sqrt{3}\right\rfloor $

แล้วค่าของ $n$ จะอยู่ห่างจากจำนวนที่อยู่ในรูป $\left\lfloor b\sqrt{3} \right\rfloor $ ที่ใกล้กับ $n$ ที่สุดอย่างมากเท่าไหร่

เราก็จะใช้อีกตัวนึงมากลบความต่างตรงนั้นได้ครับ
ขอบคุณมากครับ แต่ก็ยังไม่ได้ ขอลองต่ออีกหน่อยครับ
ตอบพร้อมอ้างอิงข้อความนี้
  #242  
Old 23 เมษายน 2015, 21:34
Beatmania's Avatar
Beatmania Beatmania ไม่อยู่ในระบบ
ลมปราณคุ้มครองร่าง
 
วันที่สมัครสมาชิก: 10 พฤษภาคม 2011
ข้อความ: 279
Beatmania is on a distinguished road
Default

ลองแสดงครับว่า ทุกๆ จำนวนเต็ม $n$ เราสามารถหาจำนวนเต็ม $b$ ที่ทำให้ $\sqrt{2}+\sqrt{3}b-2<n\leq \sqrt{2}+\sqrt{3}b$ ได้เสมอ
__________________
I'm Back

23 เมษายน 2015 21:35 : ข้อความนี้ถูกแก้ไขแล้ว 2 ครั้ง, ครั้งล่าสุดโดยคุณ Beatmania
ตอบพร้อมอ้างอิงข้อความนี้
  #243  
Old 28 เมษายน 2015, 23:16
ความรู้ยังอ่อนด้อย's Avatar
ความรู้ยังอ่อนด้อย ความรู้ยังอ่อนด้อย ไม่อยู่ในระบบ
จอมยุทธ์หน้าหยก
 
วันที่สมัครสมาชิก: 18 กันยายน 2010
ข้อความ: 175
ความรู้ยังอ่อนด้อย is on a distinguished road
Default

อ้างอิง:
ข้อความเดิมเขียนโดยคุณ Beatmania View Post
ลองแสดงครับว่า ทุกๆ จำนวนเต็ม $n$ เราสามารถหาจำนวนเต็ม $b$ ที่ทำให้ $\sqrt{2}+\sqrt{3}b-2<n\leq \sqrt{2}+\sqrt{3}b$ ได้เสมอ

ผมไม่รู้ว่ามันถูกรึเปล่าครับ ลองตรวจให้ด้วยครับ

ให้ $N= \left\lfloor\,\sqrt{3}b\right\rfloor$ สำหรับทุกๆ จำนวนเต็ม $b\in \mathbf{N} $ จะมีบาง $N\in \mathbf{N}$

เราจึงได้ $\sqrt{3}b-1 < N \leq \sqrt{3}b$

ต่อมาพิจารณา $\left\lfloor\,\sqrt{3}(b+1)\right\rfloor$ ทำคล้ายๆ ข้างบน

$N+\sqrt{3}-1\leq\sqrt{3}b+\sqrt{3}-1<\left\lfloor\,\sqrt{3}(b+1)\right\rfloor \leq \sqrt{3}b+\sqrt{3}<N+\sqrt{3}+1$

เราจึงได้ $\left\lfloor\,\sqrt{3}(b+1)\right\rfloor = N+1, N+2$

ถ้าบางจำนวนเต็ม $b$ ที่ทำให้ $\left\lfloor\,\sqrt{3}(b+1)\right\rfloor =N+1 $ ก็จะได้ 2 จำนวนที่ติดกัน

แต่ถ้า บางจำนวนเต็ม $b$ ที่ทำให้ $\left\lfloor\,\sqrt{3}(b+1)\right\rfloor =N+2 $

ซึ่งขาด $N+1$ เราหาได้จาก $N+1= \left\lfloor\,\sqrt{3}b\right\rfloor +\left\lfloor\,\sqrt{2}\right\rfloor $

เพราะฉะนั้นจำนวนนับทุกจำนวนสามารถเขียนในรูป $\left\lfloor\,\sqrt{3}b\right\rfloor+\left\lfloor\,\sqrt{2}c\right\rfloor $
ตอบพร้อมอ้างอิงข้อความนี้
  #244  
Old 28 เมษายน 2015, 23:47
Beatmania's Avatar
Beatmania Beatmania ไม่อยู่ในระบบ
ลมปราณคุ้มครองร่าง
 
วันที่สมัครสมาชิก: 10 พฤษภาคม 2011
ข้อความ: 279
Beatmania is on a distinguished road
Default

ผมว่าความรู้คุณไม่อ่อนด้อยแล้วแหละ 555

ขอชื่นชมเฉลยในกระทู้ Equation solving นะครับ สวยจริงๆ

ขอให้คุณความรู้ฯ ตั้งโจทย์ต่อเลยครับ หรือถ้าขี้เกียจตั้งก็ไปหามาก็ได้ครับ
__________________
I'm Back
ตอบพร้อมอ้างอิงข้อความนี้
  #245  
Old 29 เมษายน 2015, 12:32
ความรู้ยังอ่อนด้อย's Avatar
ความรู้ยังอ่อนด้อย ความรู้ยังอ่อนด้อย ไม่อยู่ในระบบ
จอมยุทธ์หน้าหยก
 
วันที่สมัครสมาชิก: 18 กันยายน 2010
ข้อความ: 175
ความรู้ยังอ่อนด้อย is on a distinguished road
Default

70. จงหาจำนวนนับ $x,y$ ที่ $y^2=x^3+7$ (credit: passer-by)
ตอบพร้อมอ้างอิงข้อความนี้
  #246  
Old 29 เมษายน 2015, 16:33
Pitchayut Pitchayut ไม่อยู่ในระบบ
บัณฑิตฟ้า
 
วันที่สมัครสมาชิก: 20 มกราคม 2015
ข้อความ: 352
Pitchayut is on a distinguished road
Default

ข้อนี้ไม่ยากครับ แค่กรอง modulo 9 ก็ได้แล้ว

สังเกตว่า $n^2\equiv 0, 1, 4 \mod 9$ และ $n^3\equiv -1, 0, 1 \mod 9$

ดังนั้น $y^3+7\equiv 6, 7, 8 \mod 9$ แต่ว่า $y^3+7=x^2\equiv 0, 1, 4 \mod 9$ จึงเกิดข้อขัดแย้ง นั่นคือสมการนี้ไม่มีคำตอบ

ข้อ 71. ขอให้โจทย์ diophatine ง่ายๆ ไปทำก่อนก็แล้วกัน จงแก้สมการ $x^2 +y^2=3z^2$ เมื่อ $(x, y, z)\in \mathbb{Z}^3$

29 เมษายน 2015 16:38 : ข้อความนี้ถูกแก้ไขแล้ว 1 ครั้ง, ครั้งล่าสุดโดยคุณ Pitchayut
ตอบพร้อมอ้างอิงข้อความนี้
  #247  
Old 29 เมษายน 2015, 16:35
Beatmania's Avatar
Beatmania Beatmania ไม่อยู่ในระบบ
ลมปราณคุ้มครองร่าง
 
วันที่สมัครสมาชิก: 10 พฤษภาคม 2011
ข้อความ: 279
Beatmania is on a distinguished road
Default

$y^2+1=x^3+8$

เนื่องจาก $y^2+1\equiv 1,2 mod 4$ ดังนั้น $x^3 \equiv 1,2 mod 4$

ถ้าหาก $x^3\equiv 2 mod 4$ เราได้ว่า $2|x$ ทำให้ได้ว่า $4|x^3$ ขัดแย้งกับที $x^3\equiv 2 mod 4$

ดังนั้น $x^3\equiv 1 mod 4$ ทำให้ได้ว่า $x$ เป็ขเลขคี่ ทำให้ได้ว่า $x^2 \equiv 1 mod 4$

ได้ว่า $x\equiv 1 mod 4$ ดังนั้นแล้ว $x+2 \equiv 3 mod 4$

ดังนั้น จะต้องมีจำนวนเฉพาะที่ $\equiv 3 mod 4$ ที่หาร $x+2$ ลงตัว ให้เป็น $p$ เราได้ว่า

$p|x+2 , p|x^3+8 \rightarrow p|y^2+1 \rightarrow p|(y^2)^{\frac{p-1}{2}}+1 \rightarrow p|y^{p-1}+1$

ชัดเจนว่า $(y,p)=1$ จาก Fermat little theorem จะได้ว่า $y^{p-1}\equiv 1 mod p$

ทำให้ได้ว่า $p|1+1$ ซึ่งขัดแย้งกับที่ $p\equiv 3 mod 4$ ดังนั้นจึงไม่มีคำตอบครับ

71.ชัดเจนว่า $(0,0,0)$ สอดคล้องกับสมการด้านบน และถ้าหากจำนวนใดจำนวนหนึ่งเป็น $0$ เราได้ว่าทุกจำนวนจะต้องเป็น $0$

พิจารณากรณีที่ $x,y,z\in\mathbb{N}$

เราให้ $(x,y,z)$ เป็นชุดคำตอบที่มี $gcd(x,y,z)=1$ เพราะว่าถ้าหาก $(x,y,z)$ เป็นคำตอบแล้ว $(\frac{x}{g},\frac{y}{g},\frac{z}{g})$ จะเป็นคำตอบเช่นกันโดย $g=gcd(x,y,z)$

เนื่องจาก $x^2\equiv 0,1 mod 3$ เราจะได้ว่า $x^2+y^2\equiv 0,1,2 mod 3$ ทำให้ได้ว่า $3|x,3|y$

ทำให้ได้ว่า $9|x^2+y^2$ ส่งผลให้ $9|3z^2$ ทำให้ $3|z$ ขัดแย้งกับที่สมมติว่า $gcd(x,y,z)=1$

72.จงหาค่าของ $gcd(1^{3^{3^3}}+2^{3^{3^3}}+3^{3^{3^3}},2^{3^{3^3}}+3^{3^{3^3}}+4^{3^{3^3}},3^{3^{3^3}}+4^{3^{3^3}}+5^{3^{3^3}},...)$
__________________
I'm Back

29 เมษายน 2015 16:40 : ข้อความนี้ถูกแก้ไขแล้ว 2 ครั้ง, ครั้งล่าสุดโดยคุณ Beatmania
ตอบพร้อมอ้างอิงข้อความนี้
  #248  
Old 29 เมษายน 2015, 18:16
ความรู้ยังอ่อนด้อย's Avatar
ความรู้ยังอ่อนด้อย ความรู้ยังอ่อนด้อย ไม่อยู่ในระบบ
จอมยุทธ์หน้าหยก
 
วันที่สมัครสมาชิก: 18 กันยายน 2010
ข้อความ: 175
ความรู้ยังอ่อนด้อย is on a distinguished road
Default

71. โดยไม่เสียนัยให้ $a,b,c $ เป็นจำนวนนับรวม 0

ให้ $(a,b,c)$ เป็นคำตอบที่น้อยที่สุดในเซตคำตอบของสมการ ที่ไม่ใช่ $(0,0,0)$

จากโจทย์ทำให้ $3|a^2+b^2$ ทำให้เราได้ $a=3m,b=3n$

จึงได้ $3(m^2+n^2)=c^2$

และทำให้ได้ $3|c$ ให้ $c=3k$ ทำให้ได้

$m^2+n^2=3k^2$

เราจึงได้ $(m,n,k) $ เป็นอีกหนึ่งคำตอบที่มีค่าน้อยกว่า$(a,b,c)$

โดย $m<a, n<b, k<c$ สำหรับบางจำนวนเต็มบวก $a,b,c,m,n,k$

ดังนั้นคำตอบ ของสมการคือ $(0,0,0)$
ตอบพร้อมอ้างอิงข้อความนี้
  #249  
Old 30 เมษายน 2015, 09:36
ความรู้ยังอ่อนด้อย's Avatar
ความรู้ยังอ่อนด้อย ความรู้ยังอ่อนด้อย ไม่อยู่ในระบบ
จอมยุทธ์หน้าหยก
 
วันที่สมัครสมาชิก: 18 กันยายน 2010
ข้อความ: 175
ความรู้ยังอ่อนด้อย is on a distinguished road
Default

72.

ให้มีจำนวนเฉพาะ p ที่ p หารข้อความในโจทย์

พิจารณา $p|p^{3^{3^3}}+(p+1)^{3^{3^3}}+(p+2)^{3^{3^3}}$

ดังนั้น $p|2^{3^{3^3}}+1$ และ $p|1^{3^{3^3}}+2^{3^{3^3}}+3^{3^{3^3}}$

นั่นทำให้เราได้ว่า $p|3$ เราจึงได้ว่า มีเพียงแค่ $p=3$ ตัวเดียวเท่านั้น

พิจารณา $3^k|| n^{3^{3^3}}+(n+1)^{3^{3^3}}+(n+2)^{3^{3^3}}$

1 ใน 3 ตัวนั้นต้องหาร 3 ลงอย่างน้อย 1 ตัว เราจะได้ 2 กรณีคือ

$3^k||1^{3^{3^3}}+2^{3^{3^3}}$ และ $3^k|| (3n-1)^{3^{3^3}}+(3n+1)^{3^{3^3}}$

$2^{3^{27}}+1= (2+1)(2^2-2+1)((2^{3})^2-2^3+1)((2^{3^2})^2-2^{3^2}+1)...((2^{3^{26}})^2-2^{3^{26}}+1)$

แต่เนื่องจาก $3|| (2^{3^m})^2-2^{3^m}+1$

อีกกรณีนึงก็แยกตัวประกอบแล้วก็ทำเหมือนๆ กันครับ

เพราะฉะนั้น $k=28$

$gcd(1^{3^{3^3}}+2^{3^{3^3}}+3^{3^{3^3}},2^{3^{3^3}}+4^{3^{3^3}}+5^{3^{3^3}},3^{3^{3^3}}+4^{3^{3^3}}+5^{3^{3^3}},...)=3^{28}$
ตอบพร้อมอ้างอิงข้อความนี้
  #250  
Old 28 กรกฎาคม 2015, 17:05
Pitchayut Pitchayut ไม่อยู่ในระบบ
บัณฑิตฟ้า
 
วันที่สมัครสมาชิก: 20 มกราคม 2015
ข้อความ: 352
Pitchayut is on a distinguished road
Default

ปลุกหน่อยดีกว่า
73. จงพิสูจน์ว่า $\displaystyle{\frac{a^4+1}{2b^2+2c^2+3}}$ ไม่มีทางเป็นจำนวนเต็มเมื่อ $a, b, c$ เป็นจำนวนนับ
ตอบพร้อมอ้างอิงข้อความนี้
  #251  
Old 28 กรกฎาคม 2015, 20:04
กขฃคฅฆง's Avatar
กขฃคฅฆง กขฃคฅฆง ไม่อยู่ในระบบ
บัณฑิตฟ้า
 
วันที่สมัครสมาชิก: 21 เมษายน 2015
ข้อความ: 419
กขฃคฅฆง is on a distinguished road
Default

73. สมมติเป็นจำนวนเต็ม ให้ $p$ เป็นตัวประกอบเฉพาะของ $2b^2+2c^2+3$ จาก $2b^2+2c^2+3$ เป็นจำนวนคี่จะได้ $p$ เป็นจำนวนคี่

ถ้า $p\mid a$ จะได้ $p\mid 1$ เกิดข้อขัดแย้ง ดังนั้น $p\nmid a$

ถ้า $p=4k+3$ บาง $k \in \mathbb{Z} $ จะได้

$a^{4k+2} \equiv 1 \pmod{p} $

$a^{8k+4} = a^{4(2k+1)} \equiv 1 \pmod{p} $

ถ้า $p=8k+5$ จะได้ $a^{8k+4} = a^{4(2k+1)} \equiv 1 \pmod{p} $ เหมือนกัน

จาก $p\mid a^4+1 $ และจาก $a^4+1 \mid a^{4(2k+1)}+1$ จะได้ว่า $p\mid a^{4(2k+1)}+1$

นั่นคือ $a^{4(2k+1)} \equiv -1 \pmod{p}$ ดังนั้น $1 \equiv -1 \pmod{p}$ เกิดข้อขัดแย้ง

ดังนั้นตัวประกอบเฉพาะทั้งหมดของ $2b^2+2c^2+3$ จะอยู่ในรูป $8k+1$

ทำให้ $2b^2+2c^2+3$ อยู่ในรูป $8k+1$

แต่เมื่อตรวจสอบพบว่าไม่ว่า $b,c$ จะเป็นเลขคู่หรือเลขคี่จะได้ว่า $2b^2+2c^2+3$ ไม่อยู่ในรูป $8k+1$ เกิดข้อขัดแย้ง

ดังนั้น $\dfrac{a^4+1}{2b^2+2c^2+3} $ ไม่เป็นจำนวนเต็ม
__________________
เหนือฟ้ายังมีอวกาศ
ตอบพร้อมอ้างอิงข้อความนี้
  #252  
Old 28 กรกฎาคม 2015, 20:25
กขฃคฅฆง's Avatar
กขฃคฅฆง กขฃคฅฆง ไม่อยู่ในระบบ
บัณฑิตฟ้า
 
วันที่สมัครสมาชิก: 21 เมษายน 2015
ข้อความ: 419
กขฃคฅฆง is on a distinguished road
Default

74. จงพิสูจน์ว่าทุกจำนวนนับ $a$ จะมีจำนวนนับ $n$ ซึ่งมีตัวประกอบเฉพาะ $a$ ตัว และ $n \mid 2^n+1$
__________________
เหนือฟ้ายังมีอวกาศ
ตอบพร้อมอ้างอิงข้อความนี้
  #253  
Old 29 กรกฎาคม 2015, 22:37
Thgx0312555's Avatar
Thgx0312555 Thgx0312555 ไม่อยู่ในระบบ
กระบี่ประสานใจ
 
วันที่สมัครสมาชิก: 12 สิงหาคม 2011
ข้อความ: 885
Thgx0312555 is on a distinguished road
Default

induction ว่า $2^{3^k}+1$ มีตัวประกอบเฉพาะอย่างน้อย $k$ ตัว
ขั้นฐาน $k=1$ เห็นได้ชัด
ขั้นอุปนัย จาก $2^{3^{k+1}}+1=(2^{3^k}+1)(2^{2\cdot 3^k}-2^{3^k}+1)$
และ $\gcd(2^{3^k}+1,2^{2\cdot 3^k}-2^{3^k}+1)=3$
แต่ $2^{2\cdot 3^k}-2^{3^k}+1=2^{3^k}(2^{3^k}-1)+1>3$

เพราะฉะนั้นมี $p \mid 2^{2\cdot 3^k}-2^{3^k}+1$ แต่ $p \nmid 2^{3^k}+1$
$2^{3^{k+1}}+1$ มีตัวประกอบเฉพาะอย่างน้อย $k+1$ ตัว

สามารถใช้ induction พิสูจน์คล้ายๆกันว่า $3^k \mid 2^{3^k}+1$ (ใข้ fact ที่ว่า $3 \mid 2^{2\cdot 3^k}-2^{3^k}+1$)

ให้ $p_2,p_3,...,p_k$ เป็นตัวประกอบเฉพาะของ $2^{3^k}+1$ ที่ไม่ใช่ $3$ (เห็นได้ชัดว่า $p_i \neq 2$)
$\therefore 3^{k}p_2\cdots p_k \mid 2^{3^{k}p_2\cdots p_k}+1$
__________________
----/---~Alice~ จงรับรู้ไว้ ชื่อแห่งสีสันหนึ่งเดียวที่แสดงผล
---/---- ~Blue~ นี่คือ สีแห่งความหลังอันกว้างใหญ่ของเว็บบอร์ดนี้
ตอบพร้อมอ้างอิงข้อความนี้
  #254  
Old 29 กรกฎาคม 2015, 22:41
Thgx0312555's Avatar
Thgx0312555 Thgx0312555 ไม่อยู่ในระบบ
กระบี่ประสานใจ
 
วันที่สมัครสมาชิก: 12 สิงหาคม 2011
ข้อความ: 885
Thgx0312555 is on a distinguished road
Default

75. ง่ายๆครับ จงพิสูจน์ว่าสำหรับ $x,y \in \mathbb{Z}$, $17 \mid (2x+3y) \iff 17 \mid (9x+5y)$
__________________
----/---~Alice~ จงรับรู้ไว้ ชื่อแห่งสีสันหนึ่งเดียวที่แสดงผล
---/---- ~Blue~ นี่คือ สีแห่งความหลังอันกว้างใหญ่ของเว็บบอร์ดนี้

29 กรกฎาคม 2015 22:41 : ข้อความนี้ถูกแก้ไขแล้ว 1 ครั้ง, ครั้งล่าสุดโดยคุณ Thgx0312555
ตอบพร้อมอ้างอิงข้อความนี้
  #255  
Old 30 กรกฎาคม 2015, 10:50
กขฃคฅฆง's Avatar
กขฃคฅฆง กขฃคฅฆง ไม่อยู่ในระบบ
บัณฑิตฟ้า
 
วันที่สมัครสมาชิก: 21 เมษายน 2015
ข้อความ: 419
กขฃคฅฆง is on a distinguished road
Default

75. ขาไป ให้ $x=17x_1+k_1$ และ $y=17y_1+k_2$ โดยที่ $x_1,y_1,k_1,k_2 \in \mathbb{Z} $ และ $0\leqslant k_1,k_2 \leqslant 16$ จะได้ $17\mid 2k_1+3k_2$

แทนค่าจะได้ $(k_1,k_2) = (0,0) , (7,1) , (14,2) , (4,3) , (11,4) , (1,5) , (8,6) , (15,7) , (5,8) , (12,9) , (2,10) , (9,11) , (16,12) , (6,13) , (13,14) , (3,15) , (10,16)$

ซึ่งแต่ละตัวทำให้ $17\mid 9k_1+5k_2$ ดังนั้น $17\mid 9(17x_1+k_1)+5(17y_1+k_2)$ นั่นคือ $17\mid (9x+5y)$

ขากลับ ทำเหมือนกัน
__________________
เหนือฟ้ายังมีอวกาศ

30 กรกฎาคม 2015 11:16 : ข้อความนี้ถูกแก้ไขแล้ว 4 ครั้ง, ครั้งล่าสุดโดยคุณ กขฃคฅฆง
ตอบพร้อมอ้างอิงข้อความนี้
ตั้งหัวข้อใหม่ Reply


หัวข้อคล้ายคลึงกัน
หัวข้อ ผู้ตั้งหัวข้อ ห้อง คำตอบ ข้อความล่าสุด
ปัญหาชิงรางวัลข้อที่ 23: Number Theory once more warut คณิตศาสตร์อุดมศึกษา 17 28 ธันวาคม 2011 20:38
ช่วยคิดหน่อยครับ เกี่ยวกับ Number Theory kanji ทฤษฎีจำนวน 0 08 กันยายน 2006 18:22
ปัญหาชิงรางวัลข้อที่ 5: From Number Theory Marathon warut คณิตศาสตร์อุดมศึกษา 9 17 มกราคม 2006 18:47
ปัญหา Number Theory kanji ทฤษฎีจำนวน 4 16 พฤศจิกายน 2005 20:30
ขอลองตั้งคำถามบ้างครับ (Number theory) Nay ทฤษฎีจำนวน 3 15 พฤษภาคม 2005 13:40


กฎการส่งข้อความ
คุณ ไม่สามารถ ตั้งหัวข้อใหม่ได้
คุณ ไม่สามารถ ตอบหัวข้อได้
คุณ ไม่สามารถ แนบไฟล์และเอกสารได้
คุณ ไม่สามารถ แก้ไขข้อความของคุณเองได้

vB code is On
Smilies are On
[IMG] code is On
HTML code is Off
ทางลัดสู่ห้อง


เวลาที่แสดงทั้งหมด เป็นเวลาที่ประเทศไทย (GMT +7) ขณะนี้เป็นเวลา 14:32


Powered by vBulletin® Copyright ©2000 - 2024, Jelsoft Enterprises Ltd.
Modified by Jetsada Karnpracha